Maternal Child Nursing Rasmussen Module 2 NCLEX Prep

¡Supera tus tareas y exámenes ahora con Quizwiz!

The following are nursing measures commonly offered to women in labor. Which nursing intervention probably would be most effective in applying the gate-control theory for relief of labor pain? a.Encourage the woman to rest between contractions. b.Change the woman's position. c.Give the prescribed medication. d.Massage the woman's back.

d.Massage the woman's back.

The nurse is reviewing the symptoms of a client during her pregnancy. About which client symptom does the nurse inform the primary care provider immediately? a.Dyspnea b.Persistent anemia c.Fluid leaking from the vagina d.Imbalanced nutrition

c.Fluid leaking from the vagina

A woman in early pregnancy asks you why she has palmar erythema. Your reply would be based on the principle that palmar erythema is most likely caused by which of the following? a.An increased estrogen level b.An allergy to fetal protein c.Reduced serum protein d.Chorionic gonadotropin hormone secretion

a.An increased estrogen level

Which finding from a woman's initial prenatal assessment would be considered a possible complication of pregnancy that requires reporting to a primary care provider for management? a.episodes of double vision b.increased lumbar curvature c.nasal congestion and swollen nasal membranes d.palpitations when lying on her back

a.episodes of double vision

A woman asks the nurse if she can take an over-the-counter vitamin during pregnancy rather than her prescription prenatal vitamin. A chief ingredient in prenatal vitamins that makes them important for pregnancy nutrition is: a.folic acid. b.vitamin B12. c.vitamin C. d.potassium.

a.folic acid.

True or False: Pregnant woman seem to be more susceptible to carpal tunnel syndrome than others. a.False b.True

b.True

A woman who has completed one pregnancy with a fetus (or fetuses) reaching the stage of fetal viability is called a: a.Primipara b.Primigravida c.Multipara d.Nulligravida

a.Primipara

A pregnant patient is experiencing leg cramps. What should the nurse include in the patient's teaching plan as a relief measure? (Select all that apply.) a.Avoid full leg extension. b.Elevate lower extremities. c.Elevate the legs on two pillows. d.Stand on each leg and perform a squat. e.Bend the knee and perform

a.Avoid full leg extension. b.Elevate lower extremities.

A nurse recommends to a client in labor to try concentrating intently on a photo of her family as a means of managing pain. The woman looks skeptical and asks, "How would that stop my pain?" Which explanation should the nurse give? a."It distracts your brain from the sensations of pain." b."It blocks the transmission of nerve messages of pain at the receptors." c."It causes the release of endorphins." d."It disrupts the nerve signal of pain via mechanical irritation of the nerves."

a."It distracts your brain from the sensations of pain."

A woman asks the nurse, "What protects my baby's umbilical cord from being squashed while the baby's inside of me?" The nurse's best response is: a."Your baby's umbilical cord is surrounded by connective tissue called Wharton jelly, which prevents compression of the blood vessels and ensures continued nourishment of your baby." b."Your baby's umbilical floats around in blood anyway." c."You don't need to be worrying about things like that." d."The umbilical cord is a group of blood vessels that are very well protected by the placenta."

a."Your baby's umbilical cord is surrounded by connective tissue called Wharton jelly, which prevents compression of the blood vessels and ensures continued nourishment of your baby."

The nurse is instructing a pregnant patient to consume a diet high in complete proteins. Which food item should the nurse recommend as an example of a complete protein? a.A boiled or fried egg b.Green, leafy vegetables c.A slice of whole grain toast d.Applesauce or a whole apple

a.A boiled or fried egg

A pregnant patient has a history of genital herpes lesions and has experienced outbreaks periodically throughout the pregnancy. What should the nurse instruct the patient regarding this virus if lesions are present at the time of delivery? a.A cesarean section will be advised at the time of birth. b.There are no precautions needed at the time of birth. c.The patient will need medication immediately after birth. d.The baby will be given a vaccination against the virus at birth.

a.A cesarean section will be advised at the time of birth.

The nurse is planning nutritional instructions for a pregnant patient who is a Mexican immigrant. On which areas should the nurse focus when preparing teaching for this patient? (Select all that apply.) a.Add fruits rich in vitamin C. b.Consume potatoes at every meal. c.Increase the intake of dairy products. d.Reduce the cooking time of vegetables. e.Limit the amount of added animal fat in foods.

a.Add fruits rich in vitamin C. c.Increase the intake of dairy products. d.Reduce the cooking time of vegetables. e.Limit the amount of added animal fat in foods.

A client has opted to receive epidural anesthesia during labor. Which of the following intervention should the nurse implement to reduce the risk of a significant complication associated with this type of pain management? a.Administration of 500 mL of IV Ringer's lactate b.Administration of 1000 mL of IV glucose solution c.Move the woman into a supine position d.Administration of aspirin

a.Administration of 500 mL of IV Ringer's lactate

A pregnant patient is experiencing a vaginal discharge and wants to douche. What should the nurse instruct the patient about this health practice? a.Avoid routine douching. b.Use an alkaline solution. c.Use only a commercial solution. d.Use a solution that has been chilled.

a.Avoid routine douching.

Which statement concerning neurologic and sensory development is accurate? a.Brain waves have been recorded on an electroencephalogram as early as the end of the first trimester (12 weeks). b.Fetuses respond to sound by 24 weeks and can be soothed by the sound of the mother's voice. c.Eyes are first receptive to light at 34 to 36 weeks. d.At term, the fetal brain is at least one third the size of an adult brain.

a.Brain waves have been recorded on an electroencephalogram as early as the end of the first trimester (12 weeks).

With regard to breathing techniques during labor, maternity nurses should be aware that: a.Breathing techniques in the first stage of labor are designed to increase the size of the abdominal cavity to reduce friction. b.By the time labor has begun, it is too late for instruction in breathing and relaxation. c.Controlled breathing techniques are most difficult near the end of the second stage of labor. d.The patterned-paced breathing technique can help prevent hyperventilation.

a.Breathing techniques in the first stage of labor are designed to increase the size of the abdominal cavity to reduce friction.

A patient in labor is prescribed transcutaneous electrical nerve stimulation (TENS) to help with pain relief during labor. How should the nurse explain the process of pain relief with this method? a.Counterirritation stimulation blocks pain from traveling to the spinal cord. b.Needles are inserted along meridians to release endorphins and control pain. c.A machine is used to measure the patient's ability to relax during contractions. d.Small injections of sterile saline reduce are used to reduce the amount of back pain.

a.Counterirritation stimulation blocks pain from traveling to the spinal cord.

A woman is experiencing back labor and complains of intense pain in her lower back. An effective relief measure is to use: a.Counterpressure against the sacrum. b.Pant-blow (breaths and puffs) breathing techniques. c.Effleurage. d.Biofeedback.

a.Counterpressure against the sacrum.

The physician of a patient in labor decides that an emergency cesarean birth is required to safely deliver the fetus. When preparing the operating room suite for this procedure, which medications should the nurse ensure are available for possible use? Select all that apply. a.Diazepam b.Ephedrine c.Acetaminophen d.Atropine sulfate e.Lactated Ringer's solution

a.Diazepam b.Ephedrine d.Atropine sulfate

A patient enjoys exercising and wants to know if it can continue to be done while pregnant. What should the nurse instruct the patient about exercising at this time? (Select all that apply.) a.Drink plenty of liquids to prevent dehydration. b.Limit strenuous exercise to no longer than 20 minutes. c.Eat a low-protein, simple carbohydrate snack before exercising. d.Warm up for 5 minutes by walking or cycling on low resistance. e.Avoid exercises that require jumping or rapid changes in direction.

a.Drink plenty of liquids to prevent dehydration. b.Limit strenuous exercise to no longer than 20 minutes. d.Warm up for 5 minutes by walking or cycling on low resistance. e.Avoid exercises that require jumping or rapid changes in direction.

The nurse caring for pregnant patients is identifying interventions to support the 2020 National Health Goals regarding pain relief during labor. Which interventions support these goals? Select all that apply. a.Encourage pregnant patients to prepare for childbirth by attending classes. b.Discuss the advantages of using epidural or spinal anesthesia during labor. c.Review the various opioid analgesics that can be used to control the pain of labor. d.Review the different breathing techniques that help with pain control during labor. e.Explain the various complementary and alternative therapies to help with pain control.

a.Encourage pregnant patients to prepare for childbirth by attending classes. d.Review the different breathing techniques that help with pain control during labor. e.Explain the various complementary and alternative therapies to help with pain control.

A patient in labor with chronic back pain tells the nurse about taking a dose of hydrocodone/acetaminophen (Vicodin) for labor pain prior to coming to the hospital. What should the nurse prepare to do once the fetus is delivered? a.Evaluate the fetus for withdrawal symptoms. b.Inform the physician so that liver effects can be monitored. c.Suggest that no additional narcotic pain medication be provided during labor. d.Coach the patient in breathing techniques because other pain medication is contraindicated. e.Request that the physician prescribe the same medication to be used for pain during labor.

a.Evaluate the fetus for withdrawal symptoms. b.Inform the physician so that liver effects can be monitored.

True or False: General Anesthesia is a preferred method of pain management in childbirth. a.False b.True

a.False

True or False: following her first prenatal visit, a woman will be scheduled for a sonogram to confirm pregnancy length and document healthy fetal growth at 3 to 5 weeks of pregnancy. a.False b.True

a.False

The nurse is assessing a pregnant client. The nurse finds that the client's estimated date of birth (EDB) is December 2, 2015. What would be the client's first day of the last menstrual period (LMP)? a.February 25, 2015 b.March 25, 2015 c.February 2, 2015 d.March 2, 2014

a.February 25, 2015

How does a woman who feels in control of the situation during labor influence her pain? a.Feelings of control are inversely related to the client's report of pain. b.Decreased feeling of control helps during the third stage. c.There is no association between the two factors. d.Feeling in control shortens the overall length of labor.

a.Feelings of control are inversely related to the client's report of pain.

A nurse caring for a laboring woman should know that meconium is produced by: a.Fetal intestines b.Fetal kidneys c.Amniotic fluid d.The placenta

a.Fetal intestines

A pregnant patient asks if an over-the-counter vitamin can be taken during pregnancy instead of the prescribed prenatal vitamin. What should the nurse explain as the chief ingredient in prenatal vitamins that makes them important for pregnancy nutrition? a.Folic acid b.Vitamin C c.Potassium d.Vitamin B12

a.Folic acid

During a client's physical examination, the nurse notes that the lower uterine segment is soft on palpation. The nurse would document this finding as: a.Hegar sign. b.McDonald sign. c.Chadwick sign. d.Goodell sign.

a.Hegar sign.

The nurse caring for a pregnant woman knows that her health teaching regarding fetal circulation has been effective when the woman reports that she has been sleeping: a.In a side-lying position. b.On her back with a pillow under her knees. c.With the head of the bed elevated. d.On her abdomen.

a.In a side-lying position.

Congenital disorders refer to those conditions that are present at birth. These disorders may be inherited and caused by environmental factors or maternal malnutrition. Toxic exposures have the greatest effect on development between 15 and 60 days of gestation. For the nurse to be able to conduct a complete assessment of the newly pregnant client, he or she should be knowledgeable regarding known human teratogens, which include: a.Infections b.Radiation c.Maternal conditions d.Drugs e.Chemical

a.Infections b.Radiation c.Maternal conditions d.Drugs e.Chemical

A maternity nurse should be aware of which fact about the amniotic fluid? a.It serves as a source of oral fluid and as a repository for waste from the fetus. b.The volume remains about the same throughout the term of a healthy pregnancy. c.A volume of less than 300 ml is associated with gastrointestinal malformations. d.A volume of more than 2 L is associated with fetal renal abnormalities.

a.It serves as a source of oral fluid and as a repository for waste from the fetus.

With regard to protein in the diet of pregnant women, nurses should be aware that: a.Many protein-rich foods are also good sources of calcium, iron, and B vitamins. b.Many women need to increase their protein intake during pregnancy. c.As with carbohydrates and fat, no specific recommendations exist for the amount of protein in the diet. d.High-protein supplements can be used without risk by women on macrobiotic diets.

a.Many protein-rich foods are also good sources of calcium, iron, and B vitamins.

A woman in active labor receives an opioid agonist analgesic. Which medication relieves severe, persistent, or recurrent pain, creates a sense of well-being, overcomes inhibitory factors, and may even relax the cervix but should be used cautiously in women with cardiac disease? a.Meperidine (Demerol) b.Promethazine (Phenergan) c.Butorphanol tartrate (Stadol) d.Nalbuphine (Nubain)

a.Meperidine (Demerol)

Which food below is not a good source of iron? a.Milk b.Legumes c.Grains d.Beef

a.Milk

The nurse is assessing a client with couvade syndrome. What symptoms is the nurse likely to find? Select all that apply. a.Nausea b.Skin rashes c.Sore throat d.Weight gain e.Persistent cough

a.Nausea d.Weight gain

With regard to medications, herbs, shots, and other substances normally encountered, the maternity nurse should be aware that during pregnancy: a.Prescription and over-the-counter (OTC) drugs that otherwise are harmless can be made hazardous by metabolic deficiencies of the fetus. b.The greatest danger of drug-caused developmental deficits in the fetus is seen in the final trimester. c.Killed-virus vaccines (e.g., tetanus) should not be given, but live-virus vaccines (e.g., measles) are permissible. d.No convincing evidence exists that secondhand smoke is potentially dangerous to the fetus.

a.Prescription and over-the-counter (OTC) drugs that otherwise are harmless can be made hazardous by metabolic deficiencies of the fetus.

Many parents-to-be have questions about multiple births. Maternity nurses should be able to tell them that: a.Rates of twinning and other multiple births are increasing because of the use of fertility drugs and delayed childbearing. b.Dizygotic twins (two fertilized ova) have the potential to be conjoined twins. c.Identical twins are more common in Caucasian families. d.Fraternal twins are same gender, usually male.

a.Rates of twinning and other multiple births are increasing because of the use of fertility drugs and delayed childbearing.

The nurse is planning a seminar that focuses on the 2020 National Health Goals during pregnancy for patients who are in the first trimester of pregnancy. Which information should the nurse include in this seminar? (Select all that apply.) a.Refusing alcohol b.Importance to stop smoking c.Maintaining health appointments d.Seeking alternative care approaches e.Abstaining from drugs and substances

a.Refusing alcohol b.Importance to stop smoking c.Maintaining health appointments e.Abstaining from drugs and substances

The volume of amniotic fluid is an important factor in assessing fetal well-being. Oligohydramnios (an amniotic fluid volume of less than 300 ml) is associated with what kind of fetal anomalies? a.Renal b.Cardiac c.Gastrointestinal d.Neurologic

a.Renal

Nurses should be aware of the difference that experience can make in labor pain, such as: a.Sensory pain for nulliparous women often is greater than for multiparous women during early labor. b.Affective pain for nulliparous women usually is less than for multiparous women throughout the first stage of labor. c.Women with a history of substance abuse experience more pain during labor. d.Multiparous women have more fatigue from labor and therefore experience more pain.

a.Sensory pain for nulliparous women often is greater than for multiparous women during early labor.

A pregnant woman reports that she is still playing tennis at 32 weeks of gestation. The nurse would be most concerned regarding what this woman consumes during and after tennis matches. Which is the most important? a.Several glasses of fluid b.Extra protein sources, such as peanut butter c.Salty foods to replace lost sodium d.Easily digested sources of carbohydrate

a.Several glasses of fluid

Which behavior indicates that a woman is "seeking safe passage" for herself and her infant? a.She keeps all prenatal appointments. b.She "eats for two." c.She drives her car slowly. d.She wears only low-heeled shoes.

a.She keeps all prenatal appointments.

During a previous prenatal visit, the nurse focused on the importance of adequate nutritional intake with a pregnant patient. Which assessment findings indicate that this teaching has been effective? (Select all that apply.) a.Shiny hair b.Smooth tongue c.Conjunctiva pale d.Chipped finger nails e.Normal muscle reflexes

a.Shiny hair b.Smooth tongue e.Normal muscle reflexes

At midpoint during pregnancy, you review beginning signs of labor with a patient. One of the beginning signs of labor you would review is a.a sudden gush of clear fluid from the vagina. b.excessive fatigue and headache. c.sharp, right-sided abdominal pain. d.an increased pulse rate and upper abdominal pain.

a.a sudden gush of clear fluid from the vagina.

A client and her husband have prepared for natural birth, however, as the client progresses to 8 cm dilation, she can no longer endure the pain and begs the nurse for an epidural. What is the nurse's best response? a.Support the client's decision and call the obstetrician b.Ask the husband to gently remind her of their goal of natural birth and to encourage and help her c.Gently remind the client of her goal of a natural birth and encourage and help her d.Suggest a less extreme alternative such as a sedative

a.Support the client's decision and call the obstetrician

The various systems and organs develop at different stages. Which statement is accurate? a.The cardiovascular system is the first organ system to function in the developing human. b.Hematopoiesis originating in the yolk sac begins in the liver at 10 weeks. c.The body changes from straight to C-shaped at 8 weeks. d.The gastrointestinal system is mature at 32 weeks.

a.The cardiovascular system is the first organ system to function in the developing human.

With regard to the father's acceptance of the pregnancy and preparation for childbirth, the maternity nurse should know that: a.The father goes through three phases of acceptance of his own. b.The father's attachment to the fetus cannot be as strong as that of the mother because it does not start until after birth. c.In the last 2 months of pregnancy, most expectant fathers suddenly get very protective of their established lifestyle and resist making changes to the home. d.Typically men remain ambivalent about fatherhood right up to the birth of their child.

a.The father goes through three phases of acceptance of his own.

A nurse is preparing a patient for rhythm strip testing. She places the woman into a semi-Fowler's position. What is the appropriate rationale for this measure? a.To prevent supine hypotension syndrome b.To decrease the heart rate of the fetus c.To prevent the woman from falling out of bed d.To aid the woman as she pushes during labor

a.To prevent supine hypotension syndrome

True or False: Cocaine use during pregnancy is associated with loosening of the placenta (placenta abruption) a.True b.False

a.True

True or False: Smoking may be a cause of ectopic (tubal) pregnancy. a.True b.False

a.True

True or False: some women report cravings for nonfood substances such as clay, dirt, cornstarch, or ice cubes during pregnancy. a.True b.False

a.True

What are some safety measures to take while pregnant? Select all that apply. a.Use correct body mechanics. b.Avoid travel to high-altitude regions above 1000 feet. c.Perform activities requiring coordination, balance, and concentration. d.Take rest periods, reschedule daily activities to meet rest and relaxation needs. e.Avoid environmental teratogens, such as cleaning agents, paints, sprays, herbicides, and pesticides. f.Use safety features on tools and vehicles (e.g., safety seat belts, shoulder harnesses, headrests, goggles, helmets) as specified

a.Use correct body mechanics. d.Take rest periods, reschedule daily activities to meet rest and relaxation needs. e.Avoid environmental teratogens, such as cleaning agents, paints, sprays, herbicides, and pesticides. f.Use safety features on tools and vehicles (e.g., safety seat belts, shoulder harnesses, headrests, goggles, helmets) as specified

The coach of a client in labor is holding the client's hand and appears to be intentionally applying pressure to the space between the first finger and thumb on the back of the hand. The recognizes this as which form of therapy? a.acupressure b.acupuncture c.biofeedback d.effleurage

a.acupressure

A primigravida client has just arrived in early labor and is showing signs of extreme anxiety over the birthing process. Why should the nurse prioritize helping the client relax? a.anxiety can slow down labor and decrease oxygen to the fetus b.decreased anxiety will increase trust in the nurse c.anxiety will increase blood pressure, increasing risk with an epidural d.increased anxiety will increase the risk for needing anesthesia

a.anxiety can slow down labor and decrease oxygen to the fetus

When teaching a client about nutrition during pregnancy, the nurse should include which long-term outcomes in the plan of care? Select all that apply. a.designing a diet consistent with cultural factors b.developing healthy patterns for a lifetime c.finding sources of low fat foods d.incorporating food to build bone mass e.identifying foods to build iron stores

a.designing a diet consistent with cultural factors b.developing healthy patterns for a lifetime d.incorporating food to build bone mass e.identifying foods to build iron stores

A woman in her 20s explains to the nurse that she would like to eat more healthy foods during her pregnancy but complains about the high cost of food. She confides that she just makes minimum wage at her job and that things are tight for her financially. What would be the most appropriate intervention for the nurse to make? a.discuss the WIC program with the client b.discuss the school lunch program with the client c.recommend that the client focus on purchasing and eating only fruits and vegetables, to save money d.offer some money to the client to help her with her groceries

a.discuss the WIC program with the client

A woman in her third trimester is suffering from heartburn. What should the nurse advise her to do? a.eat small meals frequently rather than large meals b.sleep on the back with the feet elevated c.lie down immediately after eating d.consume tomato products and citrus juices regularly

a.eat small meals frequently rather than large meals

A nurse is assessing a client's nutritional intake during pregnancy. What is the best method for accomplishing this? a.enacting a 24-hour nutrition recall b.having the client describe her food cravings c.weighing the client d.calculating the client's BMI

a.enacting a 24-hour nutrition recall

A woman in her third trimester shows the nurse a narrow, brown line that has formed on her abdomen, running from her belly button down to her pubic region. She expresses concern about this and asks the nurse whether it is normal. The nurse explains that this is a normal occurrence of pregnancy and that it results from the release of melanocyte-stimulating hormone from the pituitary, causing the appearance of extra pigmentation of the skin. What is the name of this phenomenon? a.linea niagra b.diastasis c.striae gravidarium d.melasma

a.linea niagra

As part of her physical examination of a pregnant client, the nurse examines the woman's breasts. Which are healthy breast changes that indicate pregnancy? Select all that apply. a.montgomery tubercles become prominent b.overall breast size increases c.areolae darken d.hard, painless lumps form e.breasts become softer in consistency f.blue streaking of veins becomes prominent

a.montgomery tubercles become prominent b.overall breast size increases c.areolae darken f.blue streaking of veins becomes prominent

A nurse counsels a pregnant woman regarding her recommended daily allowance of calories. She advises her to obtain her carbohydrate calories from complex carbohydrates rather than simple carbohydrates. What is the best rationale for this guidance? a.more consistent regulation of glucose and insulin b.provision of a greater amount of calories per gram c.greater fatty acid content d.faster digestion of complex than simple carbohydrates

a.more consistent regulation of glucose and insulin

The nurse notices that a client in labor who is receiving an epidural has suddenly become drowsy. When the nurse asks her how she is doing, the client complains of a metallic taste in her mouth and blurred vision. Her speech is slurred. The nurse recognizes this as a serious complication related to the epidural. Which intervention would the nurse implement as a priority in this situation? Select all that apply. a.prepare for prompt birth of the fetus b.administration of a narcotic c.administration of aspirin d.administration of 500 mL of IV Ringers lactate e.administer oxygen to the client f.administer an anticonvulsant to the client

a.prepare for prompt birth of the fetus e.administer oxygen to the client f.administer an anticonvulsant to the client

What areas of teaching would the nurse cover during the preconception visit? Select all that apply. a.refraining from alcohol intake b.beginning high-intensity exercise c.finding a pediatric provider d.eating nutrient-dense foods e.attending child-care classes

a.refraining from alcohol intake d.eating nutrient-dense foods

A woman is 8 months pregnant. She tells the nurse that she knows her baby listens to her, but her husband thinks she is imagining things. Which response by the nurse is most appropriate? a."Many women imagine what their baby is like." b."A baby in utero does respond to the mother's voice." c."You'll need to ask the doctor if the baby can hear yet." d."Thinking that your baby hears will help you bond with the baby."

b."A baby in utero does respond to the mother's voice."

During the health assessment of a pregnant client who is 30 years old, the nurse discusses preventative breast care. Which recommendation would the nurse include? a."Beginning now, at age 30, you should have annual mammograms." b."A breast examination by a health care provider can help detect if there are any structural problems that may affect breastfeeding." c."You should perform monthly breast self-examination." d."You should consider being vaccinated against HPV, a virus associated with the development of breast cancer."

b."A breast examination by a health care provider can help detect if there are any structural problems that may affect breastfeeding."

The nurse is teaching a group of pregnant women about using imagery as a relaxation technique. Which advice should be included to explain passive imagery? a."Imagine that you are standing on a hill top." b."Imagine a serene sea beach with white sand." c."Imagine the feel of river water on your hands." d."Imagine that you are cycling through the green countryside.

b."Imagine a serene sea beach with white sand."

A pregnant woman enjoys exercising at a local health spa once a week. Which comment would lead the nurse to believe she needs additional health teaching? a."I limit exercising to low-impact aerobics." b."Nothing feels nicer than a hot sauna after exercise." c."I'm learning to play table tennis." d."The gym gets hot and stuffy by midmorning."

b."Nothing feels nicer than a hot sauna after exercise."

A client in the first trimester of pregnancy tells the nurse, "I urinate frequently and am not able to hold urine even for a short time." What would the nurse suggest to ease the client's discomfort? a."Eat dry carbohydrates." b."Perform Kegel exercises." c."Apply local heat or ice." d."Get back rubs regularly."

b."Perform Kegel exercises."

The spouse of a pregnant patient is concerned about the risk of paralysis from an epidural block being used during labor. How should the nurse respond to the spouse's concern? a."I have never read or heard of this happening." b."The injection is given in the space outside the spinal cord." c."An injury is unlikely because of expert professional care given." d."The injection is given at the third or fourth thoracic vertebrae so paralysis is not a problem."

b."The injection is given in the space outside the spinal cord."

Which time span delineates the appropriate length for a normal pregnancy? a.9 lunar months, 8.5 calendar months, 39 weeks, 272 days b.10 lunar months, 9 calendar months, 40 weeks, 280 days c.9 calendar months, 10 lunar months, 42 weeks, 294 days d.9 calendar months, 38 weeks, 266 days

b.10 lunar months, 9 calendar months, 40 weeks, 280 days

A patient is in her 22nd week of pregnancy and is preparing to have her fundal height measured. Given the patient's stage of gestation and following McDonald's rule, what result does the nurse expect? a.11 cm b.22 cm c.2.2 cm d.44 cm

b.22 cm

A newly married couple plans to use natural family planning. It is important for them to know how long an ovum can live after ovulation. The nurse knows that teaching is effective when the couple responds that an ovum is considered fertile for: a.6 to 8 hours b.24 hours c.2 to 3 days d.1 week

b.24 hours

A nurse is serving as a doula to a client who is now in labor at an alternative birthing center. The client has opted for a water birth, and the nurse is now drawing the water into a large tub. What temperature should the nurse keep the water at? a.39 degrees Celsius b.37 degrees Celsius c.41 degrees Celsius d.35 degrees Celsius

b.37 degrees Celsius

With regard to spinal and epidural (block) anesthesia, nurses should know that: a.This type of anesthesia is commonly used for cesarean births but is not suitable for vaginal births. b.A high incidence of postbirth headache is seen with spinal blocks. c.Epidural blocks allow the woman to move freely. d.Spinal and epidural blocks are never used together.

b.A high incidence of postbirth headache is seen with spinal blocks.

A nurse is reviewing the medical records of several pregnant women who come to the center for care. Which woman would the nurse most likely identify as being at greatest risk for nutritional deficiency during pregnancy? a.A woman who never worries about dieting b.A woman with a 1-year-old son c.A woman with 10-year-old twins d.A woman who rarely eats fruit

b.A woman with a 1-year-old son

What advice should the nurse provide to a pregnant client who admits to continuing to drink alcohol 1 to 2 times a week? a.She should avoid alcohol in the first trimester. b.Alcohol should not be consumed during pregnancy. c.The effects of alcohol on the fetus are not fully understood. d.She may have an occasional drink after the first trimester.

b.Alcohol should not be consumed during pregnancy.

A woman who is 32 weeks pregnant is informed by the nurse that a danger sign of pregnancy could be: a.Constipation b.Alteration in the pattern of fetal movement c.Heart palpitations d.Edema in the ankles and feet at the end of the day

b.Alteration in the pattern of fetal movement

A pregnant client is undergoing a fetal biophysical profile. Which parameter of the profile helps measure long-term adequacy of the placental function? a.Fetal reactivity b.Amniotic fluid volume c.Fetal breathing record d.Fetal heart rate

b.Amniotic fluid volume

Which of the following would you suggest to a pregnant woman who follows a vegetarian diet? a.Include at least one serving of meat daily. b.Anticipate needing a vitamin B12 supplement during pregnancy. c.Be careful not to eat more than four servings of fruit daily. d.Discontinue a vegetarian diet for the remainder of pregnancy.

b.Anticipate needing a vitamin B12 supplement during pregnancy.

With regard to the structure and function of the placenta, the maternity nurse should be aware that: a.As the placenta widens, it gradually thins to allow easier passage of air and nutrients b.As one of its early functions, the placenta acts as an endocrine gland c.The placenta is able to keep out most potentially toxic substances, such as cigarette smoke, to which the mother is exposed d.Optimal blood circulation is achieved through the placenta when the woman is lying on her back or standing

b.As one of its early functions, the placenta acts as an endocrine gland

A nurse is teaching a group of pregnant women about the importance of good nutrition during pregnancy. After describing the B vitamins, the nurse determines that the teaching was effective when the group states which food as the best source of vitamin B12? a.Carrots b.Beef c.Bread d.Lemons

b.Beef

The nurse is assessing a pregnant client who is in the second trimester. The client tells the nurse, "My body shook for a while when I was sitting on my couch." What laboratory parameter would the nurse monitor? a.Blood glucose levels b.Blood pressure c.Complete blood cell count d.Electroencephalogram (EEG)

b.Blood pressure

With regard to nutritional needs during lactation, a maternity nurse should be aware that: a.The mother's intake of vitamin C, zinc, and protein now can be lower than during pregnancy. b.Caffeine consumed by the mother accumulates in the infant, who therefore may be unusually active and wakeful. c.Critical iron and folic acid levels must be maintained. d.Lactating women can go back to their prepregnant calorie intake.

b.Caffeine consumed by the mother accumulates in the infant, who therefore may be unusually active and wakeful.

A pregnant woman tells you that she wants to avoid saturated fat by using vegetable oil. What is another advantage of vegetable oil? a.Aids fluid balance b.Contains linoleic acid c.Stimulates kidney function d.Has a high potassium level

b.Contains linoleic acid

A woman who is 4 months pregnant has pyrosis. Which suggestion would the nurse give her? a.Take 30 mL of milk of magnesia after every meal. b.Eat small meals and do not lie down after meals. c.Try to include complex carbohydrates in meals. d.Increase vitamin intake by adding more citrus fruit.

b.Eat small meals and do not lie down after meals.

A pregnant woman experiencing nausea and vomiting should: a.Drink a glass of water with a fat-free carbohydrate before getting out of bed in the morning. b.Eat small, frequent meals (every 2 to 3 hours). c.Increase her intake of high-fat foods to keep the stomach full and coated. d.Limit fluid intake throughout the day.

b.Eat small, frequent meals (every 2 to 3 hours).

Which findings could be considered to be a barrier to a pregnant woman seeking prenatal care? Select all that apply. a.Patient would prefer to be cared for by a midwife instead of a physician. b.Economic cost of health care. c.Patient's cultural beliefs do not include prenatal care as being valued. d.Patient speaks several languages. e.Patient had a bad experience the last time she went to a doctor for care.

b.Economic cost of health care. c.Patient's cultural beliefs do not include prenatal care as being valued. e.Patient had a bad experience the last time she went to a doctor for care.

With regard to systemic analgesics administered during labor, nurses should be aware that: a.Systemic analgesics cross the maternal blood-brain barrier as easily as they do the fetal blood-brain barrier. b.Effects on the fetus and newborn can include decreased alertness and delayed sucking. c.IM administration is preferred over IV administration. d.IV patient-controlled analgesia (PCA) results in increased use of an analgesic.

b.Effects on the fetus and newborn can include decreased alertness and delayed sucking.

True or False: taking a fiber laxative is a better choice for preventing constipation than eating fiber-rich foods. a.True b.False

b.False

A nurse teaches a pregnant woman about the presumptive, probable, and positive signs of pregnancy. The woman demonstrates understanding of the nurse's instructions if she states that a positive sign of pregnancy is: a.A positive pregnancy test result. b.Fetal movement palpated by the nurse-midwife. c.Braxton Hicks contractions. d.Quickening.

b.Fetal movement palpated by the nurse-midwife.

A pregnant patient reports feeling pain similar to menstrual cramps. What should the nurse explain about this patient's symptoms? a.Exercise helps reduce the frequency of them. b.If rhythmical, they could indicate preterm labor. c.Lying down for a few hours will help them stop. d.They are false labor and do not need to be reported.

b.If rhythmical, they could indicate preterm labor.

The health care provider is reluctant to provide pain medication to a patient delivering a preterm fetus. What should the nurse explain to the patient as the reason for the preterm fetus being more affected by medication? a.Affinity of the preterm fetus to fat-soluble drugs b.Inability of the immature liver to metabolize or inactivate drugs c.Affinity of the preterm fetus to drugs that are strongly bound to protein d.Inability of the preterm fetus to use drugs with a molecular weight over 1,000

b.Inability of the immature liver to metabolize or inactivate drugs

Cardiovascular system changes occur during pregnancy. Which finding would be considered normal for a woman in her second trimester? a.Less audible heart sounds (S1, S2) b.Increased pulse rate c.Increased blood pressure d.Decreased red blood cell (RBC) production

b.Increased pulse rate

A 16-year-old girl arrives at the office for her first prenatal visit. After missing her period, she took two home pregnancy tests, both of which were positive, and came in a week ago for a blood test. The blood test also was positive. Today she appears pale and apprehensive, and asks "Is there any other kind of test I can take, just to make sure?" Which of the following would be the most accurate nursing diagnosis of this patient? a.Anxiety due to concerns about money b.Ineffective denial of pregnancy due to apprehension and fear c.Fear related to possibility of losing the pregnancy because of a previous miscarriage d.Ineffective denial of having a baby related to the absence of quickening

b.Ineffective denial of pregnancy due to apprehension and fear

Women with inadequate weight gain during pregnancy are at higher risk of giving birth to an infant with: a.Spina bifida. b.Intrauterine growth restriction. c.Diabetes mellitus. d.Down syndrome.

b.Intrauterine growth restriction.

A woman is taking vaginal progesterone suppositories during her first trimester because her body does not produce enough of it naturally. She asks the nurse what function this hormone has in her pregnancy. What should the nurse explain is the primary function of progesterone? a.Contributes to mammary gland development b.Maintains the endometrial lining of the uterus during pregnancy c.Ensures the corpus luteum of the ovary continues to produce estrogen d.Regulates maternal glucose, protein, and fat levels

b.Maintains the endometrial lining of the uterus during pregnancy

A pregnant patient is experiencing some integumentary changes and is concerned that they may represent abnormal findings. The nurse provides information to the patient that the following findings would be considered "normal abnormal" findings during pregnancy so that she should not be alarmed. Select all that apply. a.Facial edema b.Melasma c.Linea nigra d.Superficial thrombophlebitis e.Vascular spiders f.Allodynia

b.Melasma c.Linea nigra e.Vascular spiders

What type of cultural concern is the most likely deterrent to many women seeking prenatal care? a.Religion b.Modesty c.Ignorance d.Belief that physicians are evil

b.Modesty

In understanding and guiding a woman through her acceptance of pregnancy, a maternity nurse should be aware that: a.Nonacceptance of the pregnancy very often equates to rejection of the child. b.Mood swings are most likely the result of worries about finances and a changed lifestyle, as well as profound hormonal changes. c.Ambivalent feelings during pregnancy are usually seen only in emotionally immature or very young mothers. d.Conflicts such as not wanting to be pregnant or childrearing and career-related decisions need not be addressed during pregnancy because they will resolve themselves naturally after birth.

b.Mood swings are most likely the result of worries about finances and a changed lifestyle, as well as profound hormonal changes.

The nurse is assessing a pregnant client who has undergone bariatric surgery in the past. What will the nurse primarily check in the client's health records? a.Family history b.Nutritional status c.Blood glucose levels d.Blood pressure

b.Nutritional status

A pregnant patient nearing her due date expresses anxiety over the labor and delivery process. Which outcome should the nurse select as appropriate for the patient during the delivery process? a.Patient requests pain medication throughout the labor process. b.Patient uses breathing techniques to control anxiety and pain during labor. c.Patient tolerates the use of sanitary napkins to absorb vaginal secretions during labor. d.Patient refuses complementary and alternative techniques to control pain during labor.

b.Patient uses breathing techniques to control anxiety and pain during labor.

The nurse is assessing a woman who is pregnant for the first time. Which of the following terms applies to this client? a.Nulligravida b.Primigravida c.Primipara d.Multipara

b.Primigravida

After change of shift report, the nurse assumes care of a multiparous client in labor. The woman is complaining of pain that radiates to her abdominal wall, lower back, and buttocks, and down her thighs. Before implementing a plan of care, the nurse should understand that this type of pain is: a.Visceral b.Referred c.Somatic d.Afterpain

b.Referred

A teenager arrives in the emergency department in active labor having no prenatal care. Upon examination in L and D, the nurse notices a cluster of pinpoint vesicles with a red base on her vulva which the teenager asks the nurse to not touch "her rash" because it hurts when touched. At this point, which intervention should the nurse preform next? a.Put gloves on and palpate the vulva for enlarged lymph nodes b.Report this finding to the health care provider immediately so he or she can plan the birth route c.Obtain a surgical scrub brush and chlorhexidine and cleanse the area thoroughly d.Ask the teenager if she has this rash internally as well as on the outside

b.Report this finding to the health care provider immediately so he or she can plan the birth route

A client the nurse meets in a prenatal clinic states that she has a vaginal discharge and asks about douching. Which rule is safe regarding douching during pregnancy? a.Only commercial solutions should be used. b.Routine douching is not advised. c.The solution used never should be acid. d.A room-temperature solution should not be used.

b.Routine douching is not advised.

It is important for the nurse to understand that the placenta: a.Produces nutrients for fetal nutrition b.Secretes both estrogen and progesterone c.Forms a protective, impenetrable barrier to microorganisms such as bacteria and viruses d.Excretes prolactin and insulin

b.Secretes both estrogen and progesterone

A nurse trained in reflexology is working on the feet of a client who is in early labor, at the request of the client. The nurse is applying pressure to the inside ankle about halfway between the ankle bone and the heel. The client asks the nurse what effect that is likely to have. Which information would the nurse include in the response? a.Stimulation of the fetus to promote descent b.Stimulation of the uterus to speed up labor c.Stimulation of the lactiferous glands of the breasts to stimulate milk production d.Stimulation of the spine to reduce back pain

b.Stimulation of the uterus to speed up labor

The nurse is assessing the fundal height of a pregnant client. During the assessment, the nurse observes that the client has difficulty breathing and is sweating profusely. After recording the heart rate and blood pressure of the client, the nurse changes the client's position. What is the rationale for this nursing intervention? The client has: a.Excess body weight. b.Supine hypotension. c.Gestational hypertension. d.Respiratory tract infection

b.Supine hypotension.

The nurse is planning a prenatal educational program for a community health center. What information should the nurse include that supports the 2020 National Health Goals for nutrition in pregnancy? (Select all that apply.) a.Avoid foods high in fats and calories. b.Take prenatal vitamins as prescribed. c.Ensure a daily intake of foods with folic acid. d.Limit the intake of foods high in simple carbohydrates. e.Maintain adequate nutrition before becoming pregnant.

b.Take prenatal vitamins as prescribed. c.Ensure a daily intake of foods with folic acid. e.Maintain adequate nutrition before becoming pregnant.

A nurse is assessing a 25-year-old woman during a preconceptual visit. The nurse understands that anemia can be a problem that develops during pregnancy. Therefore, she conducts which of the following to establish a baseline measure related to this condition? a.Assesses for a yeast infection b.Tests hemoglobin level c.Determines her blood type d.Takes a PAP smear

b.Tests hemoglobin level

When describing he characteristics of the amniotic fluid to a pregnant woman, which would the nurse include? a.It is usually an acidic fluid b.The amount gradually fluctuates during pregnancy c.It limits fetal movement in utero d.It is composed primarily of organic substances

b.The amount gradually fluctuates during pregnancy

During labor, a pregnant patient's doula uses therapeutic touch and massage. Which outcome indicates that these approaches have been effective? a.The patient is not complaining of leg cramps. b.The patient is not requesting pain medication. c.The patient is focusing on a painting during contractions. d.The patient asks for a cold compress at the end of a contraction.

b.The patient is not requesting pain medication.

A woman who professes to be a strict vegetarian voices concerns about her ability to meet the nutritional needs of her fetus. Which concerns should be addressed in this session? a.Her diet will need to have an increase in fruits and vegetables to meet the nutritional requirements of her baby. b.The supplementation of Vitamin B12 should be addressed. c.There are not special concerns for vegans as they can easily meet the dietary requirements without violating their "no meat" rules. d.She should be advised to forgo her beliefs during the pregnancy to adequately provide the nutritional requirements for her pregnancy.

b.The supplementation of Vitamin B12 should be addressed.

The laboring client who is at 3 cm dilation and 25% effaced is asking for analgesia. The nurse explains the analgesia usually is not administered prior to the establishment of the active phase. What is the appropriate rationale for this practice? a.This would cause fetal depression in utero. b.This may prolong labor and increase complications. c.The effects would wear off before birth. d.This can lead to maternal hypertension.

b.This may prolong labor and increase complications.

True or False: Morning sickness is associated with rising levels of human chorionic gonadotrophin (hCG) and progesterone. a.False b.True

b.True

A woman of normal weight asks you what an ideal weight gain is during pregnancy. Which of the following would be your best answer? a.She should not gain over 20 lb. b.Twenty-five to thirty-five pounds is ideal. c.The amount of weight gain is not important. d.Any gain over 30 lb is ideal.

b.Twenty-five to thirty-five pounds is ideal.

Which statement about multifetal pregnancy is not accurate? a.The expectant mother often experiences anemia because the fetuses have a greater demand for iron. b.Twin pregnancies come to term with the same frequency as single pregnancies. c.The mother should be counseled to increase her nutritional intake and gain more weight. d.Backache and varicose veins are often more pronounced.

b.Twin pregnancies come to term with the same frequency as single pregnancies.

While assisting the primary health care provider, the nurse documents the subjective symptoms of pregnancy in a client. Which subjective symptom does the nurse record based on the client's statement? a.Vaginal changes b.Urinary frequency c.Breast enlargement d.Abdominal enlargement

b.Urinary frequency

The nurse instructs a pregnant client to avoid sitting for a long time and to wear loose-fitting pants. Which pregnancy discomfort is the nurse trying to ease? a.Constipation b.Varicose veins c.Supine hypotension d.Urinary tract infections

b.Varicose veins

A primigravida client in the second stage of labor has been moaning, screaming, and generally vocal throughout her labor. Her husband is distraught seeing his wife this way and asks the nurse for more pain medication for her. What is the nurse's best response? a.reassure the first-time father that his wife will be fine, and offer to stay with her while he takes a walk b.ask the client to describe the intensity of her pain on a scale of 0 to 10 c.page the obstetrician to evaluate the client's pain, and administer an appropriate increase in her pain medication d.assist the client with breathing and imagery techniques in an attempt to calm her down

b.ask the client to describe the intensity of her pain on a scale of 0 to 10

As part of the assessment of a client's health history during the first prenatal visit, the nurse is having the client complete a 24-hour recall to establish what and how much the woman is eating. In which section of the health history should the nurse record this information? a.chief concern b.day history/social profile c.demographic data d.history of past illnesses

b.day history/social profile

A client at 32 weeks' gestation receives an ultrasound that identifies intrauterine growth restriction. Which findings from the client's nutritional assessment would indicate to the nurse that additional teaching is needed? Select all that apply. a.history of gestational diabetes in previous pregnancy b.eating large quantities of empty calorie foods c.difficulty eating because of continuing nausea d.consuming 5 to 6 small meals each day e.maternal age less than 18 years

b.eating large quantities of empty calorie foods c.difficulty eating because of continuing nausea e.maternal age less than 18 years

A nurse explains to a pregnant woman the importance of consuming adequate iodine in her diet. Which of the following conditions can a deficiency in iodine lead to? a.anemia b.goiter c.hypercholesterolemia d.diminished bone density

b.goiter

If constipation is a problem for a woman during pregnancy, which measure would be best to recommend? a.mineral oil b.increased fiber intake c.stopping prenatal vitamins temporarily d.eating more meat products

b.increased fiber intake

in labor, a pregnant client asks why contractions hurt so much. Which answer should the nurse provide? a.release of endorphins in response to contractions b.lack of oxygen to the muscle fibers of the uterus due to compression of blood vessels c.blocking of nerve transmission via mechanical irritation of nerve fibers d.distraction of the brain cortex by other stimuli

b.lack of oxygen to the muscle fibers of the uterus due to compression of blood vessels

A pregnant woman works as a secretary at a large corporation. She states that she wants to take a leave of absence from work for pregnancy but is afraid she will lose seniority standing because of this. You would advise her that a.medically, it is not wise for any woman to work past the seventh month of pregnancy. b.legally, her employer cannot penalize her this way. c.ethically, she has no right to ask for special favors. d.personally, she is selfish to think of herself ahead of the baby this way.

b.legally, her employer cannot penalize her this way.

When discussing rest and sleep with a pregnant woman, the nurse would discuss which position to use for napping? a.on her stomach with a pillow under her breasts b.on her side with the weight of the uterus on the bed c.on her back with a pillow under her knees and hips d.on her back with a pillow under her head

b.on her side with the weight of the uterus on the bed

During the initial prenatal visit, a client indicates that she frequently experiences stress incontinence. Which of the following should the nurse recommend to the client to help relieve this condition? a.increase intake of water b.perform kegel exercises c.perform monthly perineal self-examination d.reduce intake of fluids

b.perform kegel exercises

During the initial obstetrical clinic visit, the nurse shares with a client of African descent that several blood studies will be drawn. What screening is performed for women of African descent because of the ethnically inherited nature of some disease? a.screening for Tay-Sachs disease b.screening for sickle-cell trait or disease c.screening for beta-thalassemia d.screening for cystic fibrosis

b.screening for sickle-cell trait or disease

A nurse urges a pregnant client at the first prenatal office visit to begin taking iron supplements immediately. What is the rationale for this intervention? a.to reduce the risk for hypertension b.to avoid anemia c.to prevent megalohemoglobinemia d. to maintain proper blood glucose levels

b.to avoid anemia

During initial examination in the labor room, the nurse notes a large amount of white, bubbly discharge around the vaginal walls that has a foul smell. The nurse suspects the woman has which type of sexually transmitted disease? a.syphilis b.trichomoniasis c.chlamydia d.gonorrhea

b.trichomoniasis

A woman in her first trimester shares with the nurse that she has been experiencing terrible nausea when she gets up in the morning. Which action should the nurse suggest? Select all that apply. a.eat two regular meals later in the day b.try eating a snack before bedtime c.suck on sour candies d.eat some saltine crackers before rising in the morning e.use a scopolamine patch f.delay breakfast until 10 or 11 am

b.try eating a snack before bedtime c.suck on sour candies d.eat some saltine crackers before rising in the morning f.delay breakfast until 10 or 11 am

A pregnant woman at 25 weeks of gestation tells the nurse that she dropped a pan last week and her baby jumped at the noise. Which response by the nurse is most accurate? a."That must have been a coincidence

babies can't respond like that." b."The fetus is demonstrating the aural reflex." c."Babies respond to sound starting at about 24 weeks of gestation." d."Let me know if it happens again; we need to report that to your midwife." ; c."Babies respond to sound starting at about 24 weeks of gestation."

A pregnant client complains of constipation. While checking the client's history, the nurse learns that the client is taking oral iron supplements. What instruction does the nurse give the client to relieve constipation? a."Drink mineral oil before going to bed." b."Take a stool softener before going to bed." c."Drink six to eight glasses of water every day." d."Discontinue taking iron supplements."

c."Drink six to eight glasses of water every day."

A pregnant client in the first trimester asks the nurse about taking medications while she is pregnant. She tells the nurse she heard that it can be harmful to the fetus if medications are taken at certain times during pregnancy. What is the best response by the nurse? a."There is no need for you to worry, you are not far enough along in your pregnancy for this to be a problem. b."You cannot drink alcohol, but you can take some medications, such as cold preparations and over-the-counter medications." c."Expsure to certain substances during the embryonic phase may be harmful to the developing fetus." d."As long as you are past 4 weeks of pregnancy, you should be able to take most medications."

c."Expsure to certain substances during the embryonic phase may be harmful to the developing fetus."

A woman states that she does not want any medication for pain relief during labor. Her primary care provider has approved this for her. What the nurse's best response to her concerning this choice? a."That's wonderful. Medication during labor is not good for the baby." b."Your health care provider is a man and has never been in labor, he may be underestimating the pain you will have." c."I respect your preference whether it is to have medication or not." d."Let me get you something for relaxation if you don't want anything for pain."

c."I respect your preference whether it is to have medication or not."

A woman refuses to have an epidural block because she does not want to have a spinal headache after birth. What would be the nurse's best response? a. "The anesthesiologist will do her best to avoid this." b."The pain relief offered will compensate for the discomfort afterward." c."Spinal headache is not a usual complication of epidural blocks." d."Your health care provider knows what is best for you."

c."Spinal headache is not a usual complication of epidural blocks."

A woman's husband expresses concern about risk of paralysis from an epidural block being given to his wife. Which would be the most appropriate response by the nurse? a."An injury is unlikely because of expert professional care given." b."I have never read or heard of this happening." c."The injection is given in the space outside the spinal cord." d."The injection is given at the third or fourth thoracic vertebrae so paralysis is not a problem."

c."The injection is given in the space outside the spinal cord."

Sally comes in for her first prenatal examination. This is her first child. She asks you (the nurse), "How does my baby get air inside my uterus?" The correct response is: a."The baby's lungs work in utero to exchange oxygen and carbon dioxide." b."The baby absorbs oxygen from your blood system." c."The placenta provides oxygen to the baby and excretes carbon dioxide into your bloodstream." d."The placenta delivers oxygen-rich blood through the umbilical artery to the baby's abdomen."

c."The placenta provides oxygen to the baby and excretes carbon dioxide into your bloodstream."

A woman's cousin gave birth to an infant with a congenital heart anomaly. The woman asks the nurse when such anomalies occur during development. Which response by the nurse is most accurate? a."We don't really know when such defects occur." b."It depends on what caused the defect." c."They occur between the third and fifth weeks of development." d."They usually occur in the first 2 weeks of development."

c."They occur between the third and fifth weeks of development."

An expectant father confides in the nurse that his pregnant wife, at 10 weeks of gestation, is driving him crazy. "One minute she seems happy, and the next minute she is crying over nothing at all. Is there something wrong with her?" The nurse's best response is: a."This is normal behavior and should begin to subside by the second trimester." b."She may be having difficulty adjusting to pregnancy, I will refer her to a counselor I know." c."This is called emotional lability and is related to hormone changes and anxiety during pregnancy. The mood swings will eventually subside as she adjusts to being pregnant." d."You seem impatient with her. Perhaps this is precipitating her behavior."

c."This is called emotional lability and is related to hormone changes and anxiety during pregnancy. The mood swings will eventually subside as she adjusts to being pregnant."

A client who is in the transition phase reports her pain medication last given 3 hours ago has worn off. She asks if she can have another dose of the meperidine. How should the nurse respond to her request? a."Since it has been over 3 hours, you should be able to have more of the medication." b."It is too early as the medication should be given only every 4 hours." c."Your stage of labor makes giving another dose unsafe." d."I will get permission from your health care provider."

c."Your stage of labor makes giving another dose unsafe."

At approximately _____ weeks of gestation, lecithin is forming on the alveolar surfaces, the eyelids open, and the fetus measures approximately 27 cm crown to rump and weighs approximately 1110 g. a.20 b.24 c.28 d.30

c.28

The measurement of lecithin in relation to sphingomyelin (L/S ratio) is used to determine fetal lung maturity. Which ratio reflects maturity of the lungs? a.1.4:1 b.1.8:1 c.2:1 d.1:1

c.2:1

A woman is 6 weeks pregnant. She has had a previous spontaneous abortion at 14 weeks of gestation and a pregnancy that ended at 38 weeks with the birth of a stillborn girl. What is her gravidity and parity according to the GTPAL system? a.2-0-0-1-1 b.2-1-0-1-0 c.3-1-0-1-0 d.3-0-1-1-0

c.3-1-0-1-0

Which hematocrit (HCT) and hemoglobin (HGB) results represent the lowest acceptable values for a woman in the third trimester of pregnancy? a.38% HCT, 14 g/dL HGB b.35% HCT, 13 g/dL HGB c.33% HCT, 11 g/dL HGB d.32% HCT, 10.5 g/dL HGB

c.33% HCT, 11 g/dL HGB

A pregnant woman tells you that she does not drink much fluid daily. How many glasses of fluid would the nurse recommend that the woman ingest daily during pregnancy? a.2 b.4 c.6 d.10

c.6

You encourage a pregnant woman to eat a diet high in complete protein. Assuming she likes all of the following foods, which of them would you recommend as a source of this for her? a.Applesauce or a whole apple b.A slice of whole grain toast c.A boiled or fried egg d.Green leafy vegetables

c.A boiled or fried egg

A woman is in labor with her second child. She knows that she will want epidural anesthesia, and she has already signed her consent form. What must the nurse do before the woman receives the epidural? a.Review the woman's medical history and laboratory results, and interview her to confirm all information is accurate and up to date. b.Place the woman in the fetal position on the table, and keep her steady so that she won't move during the procedure. c.Administer a fluid bolus through the IV line to reduce the risk of hypotension. d.Prepare a sterile field with the supplies and medications that will be needed.

c.Administer a fluid bolus through the IV line to reduce the risk of hypotension.

A pregnant patient received a narcotic analgesic 2 hours before delivery. The newborn is lethargic and difficult to arouse. What should the nurse prepare to do to help this newborn? a.Administer intravenous fluids. b.Apply oxygen and place in a heated bassinet. c.Administer naloxone hydrochloride. d.Provide tactile stimulation to encourage crying.

c.Administer naloxone hydrochloride.

The nurse is caring for a pregnant client in her first trimester with imbalanced nutrition due to nausea and vomiting. What nursing interventions will help maintain appropriate nutrition in the client? Select all that apply. a.Advise the client to rest as needed. b.Advise the client to increase fiber in her diet. c.Advise the client to consume small and frequent meals. d.Advise the client to eat dry crackers first thing in the morning. e.Advise the client to contact the primary health care provider if vomiting is severe

c.Advise the client to consume small and frequent meals. d.Advise the client to eat dry crackers first thing in the morning. e.Advise the client to contact the primary health care provider if vomiting is severe

A pregnant patient planning for labor is asking questions about pain control options. What should the nurse explain about pain control during labor? a.The physician will decide how much pain relief is needed during labor. b.Pain medication should be started immediately when contractions are thought to begin. c.Any medication should have maximum effect for the patient and minimal effect on the fetus. d.Any medication will interfere with the ability of the uterus to contract during labor and delivery.

c.Any medication should have maximum effect for the patient and minimal effect on the fetus.

What advice would you give to a woman who is 4 months pregnant and owns a cat? a.Give it away until after delivery. b.Be careful that it doesn't scratch her. c.Ask someone else to change the cat litter. d.Refrain from cleaning the cat's dish.

c.Ask someone else to change the cat litter.

A woman has just learned that she is pregnant and would like to know how soon she can find out via ultrasound the sex of her fetus. The nurse should respond with which of the following? a.At about 6 months b.Fetal gender can only be determined by analysis of the maternal serum c.At about 4 months d.At about 2 months

c.At about 4 months

The nurse is assisting the primary health care provider during a pelvic examination of a pregnant client. What does the nurse assess while performing a pelvic examination? a.Size of the uterus b.Height of the fundus c.Client's knowledge of Kegel exercises d.Tone of pelvic musculature

c.Client's knowledge of Kegel exercises

A couple is talking with a nurse about the pregnant woman's current experience with morning sickness in the first trimester. While they are talking, the nurse notes that the woman's husband appears clammy and pale. After a moment, he excuses himself to go to the restroom. When he returns, he jokes that he has "caught" his wife's morning sickness. He also remarks that her has gained weight recently. The nurse should explain to the couple that the husband is experiencing which of the following? a.Depression b.Emotional lability c.Couvade syndrome d.Narcissism

c.Couvade syndrome

When entering the second phase of labor, a patient tells the nurse that the pain is severe and is unsure if pain medication should be used. Which nursing diagnosis should the nurse use to guide the care of the patient at this time? a.Pain related to labor contractions b.Powerlessness related to the duration and intensity of labor c.Decision conflict related to the use of analgesia during labor d.Anxiety related to lack of knowledge about normal labor processes

c.Decision conflict related to the use of analgesia during labor

A woman in early pregnancy is concerned because she is nauseated every morning. Which measure would be best to help relieve this? a.Take a teaspoon of baking soda before breakfast. b.Delay toothbrushing until noon. c.Delay breakfast until midmorning. d.Take two aspirin on arising.

c.Delay breakfast until midmorning.

A pregnant patient tells the nurse that drinking enough fluids has always been a problem for her. What should the nurse counsel the patient as being an adequate daily amount of fluid to drink while pregnant? a.Two glasses b.Four glasses c.Eight glasses d.Ten glasses

c.Eight glasses

Transcutaneous electrical nerve stimulation (TENS) reduces pain by which of the following mechanisms? a.Efferent fibers are blocked by continuously applied high-intensity stimulation. b.Pain is prevented from traveling from the uterus to spinal cord synapses. c.Electrical impulses are created that interfere with nerve transmission. d.TENS reduces apprehension and thereby complements narcotic action.

c.Electrical impulses are created that interfere with nerve transmission.

The nurse is assessing a pregnant client and finds that the client has had spinal surgery. What does the nurse interpret from the assessment? a.The client may have higher chances of preterm delivery. b.Cesarean birth should be recommended for the client. c.Epidural anesthesia is contraindicated in the client. d.The client may have right lower quadrant pain during pregnancy

c.Epidural anesthesia is contraindicated in the client.

A pregnant woman demonstrates understanding of the nurse's instructions regarding relief of leg cramps if she: a.Wiggles and points her toes during the cramp. b.Applies cold compresses to the affected leg. c.Extends her leg and dorsiflexes her foot during the cramp. d.Avoids weight bearing on the affected leg during the cramp.

c.Extends her leg and dorsiflexes her foot during the cramp.

A pregnant woman has been diagnosed with oligohydramnios. Which presentation would the nurse suspect to find on physical examination? a.Fetus is in a breech position b.FHR baseline is within normal range c.Fetus with possible renal problems d.Increased fundal height

c.Fetus with possible renal problems

Which presumptive sign (felt by woman) or probable sign (observed by the examiner) of pregnancy is not matched with another possible cause(s)? a.Amenorrhea—stress, endocrine problems b.Quickening—gas, peristalsis c.Goodell sign—cervical polyps d.Chadwick sign—pelvic congestion

c.Goodell sign—cervical polyps

With regard to what might be called the tactile approaches to comfort management, nurses should be aware that: a.Either hot or cold applications may provide relief, but they should never be used together in the same treatment. b.Acupuncture can be performed by a skilled nurse with just a little training. c.Hand and foot massage may be especially relaxing in advanced labor, when a woman's tolerance for touch is limited. d.Therapeutic touch (TT) uses handheld electronic stimulators that produce sympathetic vibrations.

c.Hand and foot massage may be especially relaxing in advanced labor, when a woman's tolerance for touch is limited.

A pregnant patient is concerned that she is allergic to something because her hands have been red and itchy since becoming pregnant. What should the nurse explain as the cause of the patient's symptoms? a.Allergy to fetal protein b.Reduced serum protein c.Increased estrogen level d.Chorionic gonadotropin hormone secretion

c.Increased estrogen level

When teaching the pregnant patient about self-medicating for pain during labor, why did the nurse instruct the patient to avoid taking acetylsalicylic acid (aspirin)? a.Development of respiratory depression in the newborn b.Interference with the ability to concentrate on contractions c.Interference with blood coagulation with increased risk of bleeding in mother or infant d.Competition with bilirubin-binding sites in fetal circulation increases risk of kernicterus.

c.Interference with blood coagulation with increased risk of bleeding in mother or infant

Why should a woman be cautioned against taking acetylsalicylic acid (aspirin) to relieve pain in labor? a.Competition with bilirubin-binding sites in fetal circulation increases risk of kernicterus b.Development of respiratory depression in the newborn c.Interference with blood coagulation with increased risk of bleeding in mother or infant d.Interference with the ability to concentrate on contractions

c.Interference with blood coagulation with increased risk of bleeding in mother or infant

Which minerals and vitamins are usually recommended to supplement a pregnant woman's diet? a.Fat-soluble vitamins A and D b.Water-soluble vitamins C and B6 c.Iron and folate d.Calcium and zinc

c.Iron and folate

A client in the second trimester of pregnancy reports constipation. What does the nurse explain to the client as the reason for this condition? a.Progesterone levels decrease gastric acid secretions. b.Progesterone levels increase gastrointestinal (GI) motility in pregnant women. c.Iron supplements may cause constipation and darkened stool. d.Constipation is caused by inadequate carbohydrate intake.

c.Iron supplements may cause constipation and darkened stool.

With regard to the development of the respiratory system, maternity nurses should be aware that: a.The respiratory system does not begin developing until after the embryonic stage b.The infant's lungs are considered mature when the L/S ratio is 1:1, at about 32 weeks c.Maternal hypertension can reduce maternal-placental blood flow, accelerating lung maturity d.Fetal respiratory movements are not visible on ultrasound scans until at least 16 weeks

c.Maternal hypertension can reduce maternal-placental blood flow, accelerating lung maturity

A 39-year-old woman is pregnant with her first child and appears to be thrilled about it. Now in her second trimester, she talks enthusiastically with the nurse about the latest maternity clothes she has bought and models them for the nurse. She also discusses the latest trends in health foods, which she has adapted since learning of her pregnancy. The nurse interprets this information as reflecting which primary emotional response to pregnancy? a.Introversion b.Emotional lability c.Narcissism d.Stress

c.Narcissism

The nurse is preparing materials to instruct a pregnant patient about the use of a local anesthetic to block specific nerve pathways. About which type of pain reduction technique will the nurse instruct the patient? a.General anesthesia b.Pressure anesthesia c.Regional anesthesia d.Pudendal nerve block

c.Regional anesthesia

The nurse is reviewing the lab reports of a client who is 10 weeks pregnant and has a family history of diabetes mellitus. The nurse finds that the client's 1-hour glucose tolerance test is normal. What does the nurse advise the client? a.Undergo a renal function test b.Increase food intake c.Repeat the test at 28 weeks d.Undergo a 3-hour glucose test

c.Repeat the test at 28 weeks

A woman is concerned that orgasm will be harmful during pregnancy. Which statement is factual? a.Most women do not experience orgasm during pregnancy. b.Orgasm during pregnancy is potentially harmful. c.Some women experience orgasm intensely during pregnancy. d.Venous congestion in the pelvis makes orgasm painful.

c.Some women experience orgasm intensely during pregnancy.

The nurse is reviewing the signs of labor with a patient entering the last phase of the third trimester of pregnancy. What should the nurse include as an indication that the labor is beginning? a.Excessive fatigue and headache b.Sharp, right-sided abdominal pain c.Sudden gush of clear fluid from the vagina d.An increased pulse rate and upper abdominal pain

c.Sudden gush of clear fluid from the vagina

At a prenatal appointment, a woman who is 3 months pregnant confides to you that she ingests starch because of a craving she has had since adolescence. She is now 26 years old. What would be your most appropriate response? a.Arrange for a separate appointment to prepare instructions about the hazards of ingesting nonfood substances. b.Emphasize the protein, vitamin, and iron needs of pregnancy nutrition. c.Suggest she have a hemoglobin assessment done because of the association between pica and iron-deficiency anemia. d.Kindly but firmly scold her to discourage the habit.

c.Suggest she have a hemoglobin assessment done because of the association between pica and iron-deficiency anemia.

A pregnant patient who works as a secretary at a large corporation wants to take a leave of absence from work but is afraid of losing seniority. What should the nurse advise the patient? a.The patient should not ask for special favors. b.The leave of absence should occur after the baby is born. c.The employer cannot penalize the patient for being pregnant. d.It is not wise for any woman to work past the seventh month of pregnancy.

c.The employer cannot penalize the patient for being pregnant.

A woman at 35 weeks of gestation has had an amniocentesis. The results reveal that surface-active phospholipids are present in the amniotic fluid. The nurse is aware that this finding indicates that: a.The fetus is at risk for Down syndrome. b.The woman is at high risk for developing preterm labor. c.The lungs are mature. d.Meconium is present in the amniotic fluid.

c.The lungs are mature.

A woman who is 10-weeks pregnant comes to the health center for an evaluation. As part of the interview, the nurse assesses how the woman is adjusting to the pregnancy. The nurse would most likely expect the woman to display which emotional response? a.narcissism b.introversion c.ambivalence d.acceptance

c.ambivalence

Which change related to the vital signs is expected in pregnant women? a.pulse decreases b.temperature decrease c.blood pressure decreases d.lung space increases

c.blood pressure decreases

A young woman in her first trimester confesses to the nurse when questioned that she is probably not consuming enough calories. The nurse should explain to this client that deficient nutrition can hinder the baby's growth, which at this point in her pregnancy is primarily via an increase in the number of cells formed. This type of growth is known as which of the following? a.hypercholesterolemia b.hypertrophy c.hyperplasia d.hyperemesis gravidarum

c.hyperplasia

A woman in the third trimester of her first pregnancy expresses fear about the birth canal being wide enough for her to push the baby through it during labor. She is a petite person, and the baby seems so large. She asks the nurse how this will be possible. To help alleviate the client's fears, the nurse should mention the role of the hormone that softens the cervix and collagen in the joints, which allows dilation and enlargement of the birth canal. What is this hormone? a.human placental lactogen b.estrogen c.relaxin d.progesterone

c.relaxin

The nurse is assessing a primipara's fundal height at 36 weeks gestation and notes the fundus is now located at the xiphoid process of the sternum. The client asks if this is normal. Which response to the client would be best? a."To be honest, the fundus should be lower since you have gained minimal weight." b."Just get prepared, the fundus might actually get a little higher until a few days before you go into labor." c."By this time, the fundus should drop down lower because the baby is moving towards the pelvic inlet." d."At 36 weeks' gestation, the fundus is in the normal expected location."

d."At 36 weeks' gestation, the fundus is in the normal expected location."

A woman who is 14 weeks pregnant tells the nurse that she always had a glass of wine with dinner before she became pregnant. She has abstained during her first trimester and would like to know whether it is safe for her to have a drink with dinner now. The nurse tells her: a."Because you're in your second trimester, there's no problem with having one drink with dinner." b."One drink every night is too much. One drink three times a week should be fine." c."Because you're in your second trimester, you can drink as much as you like." d."Because no one knows how much or how little alcohol it takes to cause fetal problems, the best course is to abstain throughout your pregnancy."

d."Because no one knows how much or how little alcohol it takes to cause fetal problems, the best course is to abstain throughout your pregnancy."

The nurse is teaching a pregnant client about body mechanics to decrease discomfort related to the lumbar curve of pregnancy. Which statement made by the client indicates the need for additional teaching? a."I should avoid platform shoes and high heels." b."I should use a pillow in the car to support my lower back area." c."I should change positions often if I have to stand for a long time." d."I should adjust my car seat such that my knees are lower than my hips.

d."I should adjust my car seat such that my knees are lower than my hips.

The nurse is providing exercise tips to an 18-week pregnant client. Which statement made by the client indicates the need for additional teaching? a."I should exercise regularly for 30 minutes at a time." b."I should decrease weight-bearing exercises." c."I should take my pulse every 15 minutes while exercising." d."I should lie on my back for 10 minutes after exercising."

d."I should lie on my back for 10 minutes after exercising."

A pregnant woman has developed varicosities. Which statement would suggest she needs additional health teaching? a."I maintain a high fluid intake." b."I'll try not to stand for long periods." c."I dorsiflex my feet and ankles frequently." d."I wear knee-highs rather than pantyhose."

d."I wear knee-highs rather than pantyhose."

During a preconception visit, the nurse teaches the client about the risk of inadequate nutrition in early pregnancy. Which statement by the client indicates understanding of the nurse's teaching? a."If I have inadequate nutrients in the last half of pregnancy, the blood flow to the placenta will be decreased." b."If I have inadequate nutrients in the last half of pregnancy, my fetus will develop cellular hyperplasia." c."If I have inadequate nutrients in the first half of pregnancy, the fetal cells are unable to grow to their full potential." d."If I have inadequate nutrients in the first half of pregnancy, an inadequate number of fetal cells are formed."

d."If I have inadequate nutrients in the first half of pregnancy, an inadequate number of fetal cells are formed."

A pregnant patient enjoys exercising at a local health spa once a week. Which patient comment indicates to the nurse that additional health teaching is needed? a."I'm learning to play table tennis." b."I limit exercising to low-impact aerobics." c."The gym gets hot and stuffy by midmorning." d."Nothing feels nicer than a hot tub soak after exercise."

d."Nothing feels nicer than a hot tub soak after exercise."

A woman who is 8 months pregnant asks the nurse, "Does my baby have any antibodies to fight infection?" The most appropriate response by the nurse is: a."Your baby has all the immunoglobulins necessary: IgG, IgM, and IgA." b."Your baby won't receive any antibodies until he is born and you breastfeed him." c."Your baby does not have any antibodies to fight infection." d."Your baby has IgG and IgM immunoglobulins."

d."Your baby has IgG and IgM immunoglobulins."

A client in labor has requested the administration of narcotics to reduce pain. At 2 cm cervical dilation, she says that she is managing the pain well at this point but does not want it to get ahead of her. What should the nurse do? a.Agree with the client, and administer the drug immediately to keep the pain manageable b.Refuse to administer narcotics because they can develop dependency in the client and the fetus c.Explain to the client that narcotics should only be administered an hour or less before birth d.Advise the client to hold out a bit longer, if possible, before administration of the drug to prevent slowing labor

d.Advise the client to hold out a bit longer, if possible, before administration of the drug to prevent slowing labor

What is the most effective way for a nurse to assess a woman's usual food intake during her pregnancy? a.Assess a list she makes describing a good diet. b.Ask her to describe her total intake for a week. c.Assess her skin for hydration and color. d.Ask her to describe her intake for the last 24 hours.

d.Ask her to describe her intake for the last 24 hours.

A woman from Brazil and her husband are visiting the office for a prenatal checkup. Which of the following would be the best nursing intervention to show respect to the couple and their cultural and individual values related to childbirth? a.Explain to the couple how care for pregnancy has evolved from an illness approach to a view of pregnancy as healthy b.Mention to the couple several example of Brazilian birth practices that you have learned about online c.Relate your own personal birth story to the couple d.Ask the couple whether there is anything they believe should or should not be done to make the pregnancy successful

d.Ask the couple whether there is anything they believe should or should not be done to make the pregnancy successful

The nurse is preparing to assess the nutritional status of a patient who is 8 weeks pregnant. What is the most effective way for the nurse to assess the patient's food intake thus far in the pregnancy? a.Assess skin status for hydration and color. b.Ask the patient to describe total intake for a week. c.Assess a list that the patient describes as a good diet. d.Ask the patient to describe intake for the last 24 hours.

d.Ask the patient to describe intake for the last 24 hours.

The _____ is/are responsible for oxygen and carbon dioxide transport to and from the maternal bloodstream. a.Decidua basalis b.Blastocyst c.Germ layer d.Chorionic villi

d.Chorionic villi

In order to reassure and educate pregnant clients about changes in their blood pressure, maternity nurses should be aware that: a.A blood pressure cuff that is too small produces a reading that is too low, a cuff that is too large produces a reading that is too high. b.Shifting the client's position and changing from arm to arm for different measurements produces the most accurate composite blood pressure reading at each visit. c.The systolic blood pressure increases slightly as pregnancy advances, the diastolic pressure remains constant. d.Compression of the iliac veins and inferior vena cava by the uterus contributes to hemorrhoids in the later stage of term pregnancy.

d.Compression of the iliac veins and inferior vena cava by the uterus contributes to hemorrhoids in the later stage of term pregnancy.

When counseling a client about getting enough iron in her diet, the maternity nurse should tell her that: a.Milk, coffee, and tea aid iron absorption if consumed at the same time as iron. b.Iron absorption is inhibited by a diet rich in vitamin C. c.Iron supplements are permissible for children in small doses. d.Constipation is common with iron supplements.

d.Constipation is common with iron supplements.

In the current practice of childbirth preparation, emphasis is placed on: a.The Dick-Read (natural) childbirth method. b.The Lamaze (psychoprophylactic) method. c.The Bradley (husband-coached) method. d.Encouraging expectant parents to attend childbirth preparation in any or no specific method.

d.Encouraging expectant parents to attend childbirth preparation in any or no specific method.

Over-the-counter (OTC) pregnancy tests usually rely on which technology to test for human chorionic gonadotropin (hCG)? a.Radioimmunoassay b.Radioreceptor assay c.Latex agglutination test d.Enzyme-linked immunosorbent assay (ELISA)

d.Enzyme-linked immunosorbent assay (ELISA)

A patient is to have an amniocentesis with ultrasound. What does the nurse explain to the patient that amniocentesis can determine? Select all that apply. a.What the eye color of the baby will be b.How much the fetus will weight at birth c.What type of facial features the fetus will have d.For genetic testing e.Whether the fetal lungs are mature enough to support respiration outside of the womb

d.For genetic testing e.Whether the fetal lungs are mature enough to support respiration outside of the womb

A nurse is helping a patient prepare for ultrasound examination. Which intervention would be most appropriate for the nurse to implement? a.Have the father of the child leave the room to avoid x-ray exposure b.Place a towel under her left buttock c.Have the woman void before the procedure d.Have the patient drink several full glasses of water before the procedure

d.Have the patient drink several full glasses of water before the procedure

A woman who is 4 months pregnant notices frequent heart palpitations and leg cramps. She is anxious to learn how to alleviate these. Which nursing diagnosis would best apply to her? a.Impaired urinary elimination related to inability to excrete creatine from her muscles b.Risk for ineffective breathing pattern related to pressure of the growing uterus c.Pain related to severe complications of pregnancy d.Health-seeking behaviors related to ways to relieve discomforts of pregnancy

d.Health-seeking behaviors related to ways to relieve discomforts of pregnancy

A woman in the active phase of the first stage of labor is using a shallow pattern of breathing, which is about twice the normal adult breathing rate. She starts to complain about feeling lightheaded and dizzy and states that her fingers are tingling. The nurse should: a.Notify the woman's physician. b.Tell the woman to slow the pace of her breathing. c.Administer oxygen via a mask or nasal cannula. d.Help her breathe into a paper bag.

d.Help her breathe into a paper bag.

Which suggestion about weight gain is not an accurate recommendation? a.Underweight women should gain 12.5 to 18 kg. b.Obese women should gain at least 7 kg. c.Adolescents are encouraged to strive for weight gains at the upper end of the recommended scale. d.In twin gestations, the weight gain recommended for a single fetus pregnancy should simply be doubled.

d.In twin gestations, the weight gain recommended for a single fetus pregnancy should simply be doubled.

Which of the following supports why a preterm fetus usually is more affected by medication given at birth than a full-term fetus? a.Affinity of the preterm fetus to drugs that are fat-soluble b.Affinity of the preterm fetus to drugs that are strongly bound to protein c.Inability of the preterm fetus to use drugs with a molecular weight over 1000 d.Inability of the immature liver to metabolize or inactivate drugs

d.Inability of the immature liver to metabolize or inactivate drugs

An expectant couple asks the nurse about intercourse during pregnancy and whether it is safe for the baby. The nurse should tell the couple that: a.Intercourse should be avoided if any spotting from the vagina occurs afterward. b.Intercourse is safe until the third trimester. c.Safer-sex practices should be used once the membranes rupture. d.Intercourse and orgasm are often contraindicated if a history or signs of preterm labor are present.

d.Intercourse and orgasm are often contraindicated if a history or signs of preterm labor are present.

A pregnant patient is planning travel to a foreign country as part of a work assignment and needs immunizations. What should the nurse instruct the patient about immunizations while pregnant? a.Immunizations should be restricted to live viruses only. b.There are no restrictions on immunizations while pregnant. c.The only immunization that should be avoided is for the flu. d.Live virus immunizations are contraindicated while pregnant.

d.Live virus immunizations are contraindicated while pregnant.

Which developmental finding is accurate with regard to fetal growth? a.Heart starts beating at 12 weeks. b.Lungs take shape by 8 weeks. c.Brain configuration is complete by 8 weeks. d.Main blood vessels form by 8 weeks.

d.Main blood vessels form by 8 weeks.

The nurse is emphasizing the importance of adequate rest and sleep with a pregnant patient. Which position should the nurse suggest the patient use? a.On the back with a pillow under the head b.On the stomach with a pillow under her breasts c.On the back with a pillow under the knees and hips d.On the side with the weight of the uterus on the bed

d.On the side with the weight of the uterus on the bed

The nurse has identified the diagnosis of imbalanced nutrition for a pregnant patient. Which assessment data did the nurse use to identify this diagnosis for the patient? a.Patient eats salads at least twice a day. b.Patient does not like potatoes or bread. c.Patient eats red meat several times a week. d.Patient does not want to gain any weight while pregnant.

d.Patient does not want to gain any weight while pregnant.

The nurse provides instructions to a patient with hyperemesis gravidarum. Which outcome indicates that teaching has been effective? a.Patient has vomiting episodes only in the morning. b.Patient is able to tolerate soft foods after episodes of vomiting. c.Patient is able to ingest clear liquids between episodes of vomiting. d.Patient is able to ingest a regular diet after progressing through clear liquids and soft foods.

d.Patient is able to ingest a regular diet after progressing through clear liquids and soft foods

The nurse advises the woman who wants to have a nurse-midwife provide obstetric care that: a.She will have to give birth at home. b.She must see an obstetrician as well as the midwife during pregnancy. c.She will not be able to have epidural analgesia for labor pain. d.She must be having a low-risk pregnancy.

d.She must be having a low-risk pregnancy.

A pregnant patient is concerned that orgasm will be harmful to the developing fetus. What should the nurse include when responding to this patient's concern? a.Orgasm during pregnancy is potentially harmful. b.Venous congestion in the pelvis makes orgasm painful. c.Most women do not experience orgasm during pregnancy. d.Some women experience orgasm intensely during pregnancy.

d.Some women experience orgasm intensely during pregnancy.

Which possible outcome would be a major disadvantage of any pain relief method that also affects awareness of the mother? a.The father's coaching role may be disrupted at times. b.The infant may show increased drowsiness. c.The mother may have continued memory loss postpartum. d.The mother may have difficulty working effectively with contractions.

d.The mother may have difficulty working effectively with contractions.

A 22-year-old woman pregnant with a single fetus had a preconception body mass index (BMI) of 24. When she was seen in the clinic at 14 weeks of gestation, she had gained 1.8 kg (4 lbs) since conception. How would the nurse interpret this finding? a.This weight gain indicates possible gestational hypertension. b.This weight gain indicates that the woman's infant is at risk for intrauterine growth restriction (IUGR). c.This weight gain cannot be evaluated until the woman has been observed for several more weeks. d.The woman's weight gain is appropriate for this stage of pregnancy.

d.The woman's weight gain is appropriate for this stage of pregnancy.

The nurse instructs the client about skin massage and the gate-control theory of pain. Which statement would be appropriate for the nurse to include for client understanding of the nonpharmacologic pain relief methods? a.The gate control mechanism is located at the pain site. b.Pain perception is decreased if anxiety is present. c.The gate control mechanism opens so all the stimuli pass through to the brain. d.This is a technique to prevent the painful stimuli from entering the brain.

d.This is a technique to prevent the painful stimuli from entering the brain.

A nurse is preparing to perform a physical examination of a pregnant woman at her first prenatal visit. Which of the following actions should the nurse perform before beginning the physical examination? a.ask the woman to perform kegel exercises b.have the woman perform a breast self-examination c.have the woman perform a perineal self-examination d.ask the woman to void for a clean-catch urine specimen

d.ask the woman to void for a clean-catch urine specimen

A pregnant client mentions to the nurse that a friend has given her a variety of herbs to use during her upcoming labor to help manage pain. Specifically, she gave her chamomile tea, raspberry leaf tea, skullcap, catnip, jasmine, lavender, and black cohosh. Which of these should the nurse encourage the client to not take because of the risk of acute toxic effects such as cerebrovascular accident? a.jasmine b.catnip c.skullcap d.black cohosh

d.black cohosh

A pregnant woman experiences frequent leg cramps. Which measure would the nurse include in her teaching plan to provide her with relief? a.elevating her leg on two pillows b.bending her knee and dorsiflexing her foot c.plantar flexing her foot and wiggling her toes d.extending her knee and dorsiflexing her foot

d.extending her knee and dorsiflexing her foot

A woman relates to the nurse that she understands that dietary fat is bad for her and that she should avoid it during pregnancy. How should the nurse respond? a.fats are essential during pregnancy, and fish such as marlin and orange roughy are good sources b.fats are not essential during pregnancy and thus are optional c.fats should be avoided during pregnancy d.fats are essential during pregnancy, and vegetable oils are a good source

d.fats are essential during pregnancy, and vegetable oils are a good source

A woman in her 15th week of pregnancy is about to undergo amniocentesis. Which nursing intervention should be made first? a.place the client in the supine position b.observe the fetal heart rate monitor c.have the client void d.obtain a signed consent form

d.obtain a signed consent form

The injection of a local anesthetic to block specific nerve pathways is referred to as: a.amnesic medication. b.gas administration. c.natural anesthesia. d.regional block.

d.regional block.

A woman whose body mass index (BMI) is 32 has just learned that she is pregnant. Which of the following should the nurse advise her regarding her weight over the course of her entire pregnancy? a.she should gain 5 to 10 lb b.she should strive to keep her weight constant c.she should loose 5 to 10 lb d.she should gain 11 to 20 lb

d.she should gain 11 to 20 lb

An adolescent who is pregnant asks the nurse which sport would be safe for her to learn during pregnancy. Which activity would the nurse suggest as safe? a.bicycling b.jogging c.skiing d.swimming

d.swimming

A client at 29-weeks gestation tells the nurse she is experiencing aches in her hips and joints. What would the nurse do next? a.document these finding in the client's chart b.ask the client if there is a family history of arthritis c.have the primary health care provider see the client next d.tell the client these are normal findings during pregnancy

d.tell the client these are normal findings during pregnancy

An expectant mother in week 30 of her pregnancy reports to the nurse that she has been doing her kick counts several times a day and the fetus has been kicking at a rate of about 4 times per hour, on average. The nurse interprets this finding as suggesting which situation? a.the fetus is healthy b.the mother is not performing the counts correctly c.the mother has been too active d.the fetus is not receiving enough nutrients

d.the fetus is not receiving enough nutrients

Immediately following an epidural block, a woman's blood pressure suddenly falls to 90/50. The nurse's first action would be to: a.raise her head off the bed. b.ask her to inhale deeply at least five times. c.administer oxygen by facemask. d.turn her on her left side or raise her legs.

d.turn her on her left side or raise her legs.

A client who has just given a blood sample for pregnancy testing in the health care provider's office asks the nurse what method of confirming pregnancy is the most accurate. The nurse explains the difference between presumptive symptoms, probable signs, and positive signs. What should the nurse mention as an example of a positive sign, which may be used to diagnose pregnancy? a.absence of a period b.laboratory test of a blood serum specimen for hCG c.laboratory test of a urine specimen for hCG d.visualization of the fetus by ultrasound

d.visualization of the fetus by ultrasound

A pregnant woman at 10 weeks of gestation jogs three or four times per week. She is concerned about the effect of exercise on the fetus. The nurse should tell her: a."You don't need to modify your exercising any time during your pregnancy." b."Stop exercising, because it will harm the fetus." c."You may find that you need to modify your exercise to walking later in your pregnancy, around the seventh month." d."Jogging is too hard on your joints

switch to walking now." ; c."You may find that you need to modify your exercise to walking later in your pregnancy, around the seventh month."

In reviewing the history of a woman who wants to become pregnant, which medication profile would indicate a potential concern relative to toxic exposure? Select all that apply. a.Tylenol OTC occasionally for a headache

twice last week b.Anticonvulsant for seizure disorder c.Lithium for bipolar disorder d.Coumadin for atrial fibrillation e.Multivitamins once a day ; b.Anticonvulsant for seizure disorder c.Lithium for bipolar disorder d.Coumadin for atrial fibrillation


Conjuntos de estudio relacionados

Chapter 10: Service as the Offering

View Set

Ch20 The Baroque in Northern Europe

View Set

Six Sigma Practice Test 8 (Indiana Council--Measure & Analyze Stage)

View Set

Chapter 38:, GI, safety, 309 Quiz 4

View Set

What is a Pandemic and the History of Pandemics

View Set